¿Cuáles son las propiedades de transformación reales de los espinores de Dirac uσ(p)uσ(p)u_\sigma(p)?

Dejar tu σ ( pag ) Sea un espinor de Dirac . Hasta donde yo sé, se transforma bajo cambios de marco de referencia según

(1) tu σ ( pag ) = S ( Λ ) tu σ ( Λ pag )
donde el σ la etiqueta no se mezcla. ¿Por qué es esto? ¿No deberían las polarizaciones ser dependientes del marco? Después de todo, el "eje de cuantificación de giro" depende del marco.

Dicho de otra manera: la relación anterior es equivalente a

(2) tu ( Λ ) | pag , σ = | Λ pag , σ
sin σ mezclando Para mí, los estados de una partícula deberían transformarse de acuerdo con
(3) tu ( Λ ) | pag , σ = ? σ D σ σ ( Λ ) | Λ pag , σ
aunque obviamente este no es el caso. Si esto fuera cierto, entonces tendríamos
(4) tu σ ( pag ) = ? S ( Λ ) σ D σ σ ( Λ ) tu σ ( Λ pag )
en lugar de ( 1 ) .

Pregunta : ¿Por qué las polarizaciones no se mezclan bajo las transformaciones de Lorentz (en ninguna ( 1 ) ni ( 2 ) )?


EDITAR

Como señaló Blazej , los componentes de espín σ mezcle bajo la transformación de Lorentz, y la ley correcta es

tu σ ( pag ) = σ METRO σ σ tu σ ( Λ pag )
para alguna matriz METRO (que en realidad está relacionado con una pequeña matriz de grupo de Wigner, pero cuya forma no es tan relevante para mí; la parte importante es que el σ mezcla de componentes, y no cuál es la matriz real que los mezcla).

Mi preocupación es que esto no es lo que encuentro en línea: por ejemplo, vea esta respuesta en física.SE (última ecuación). Además, consulte este artículo de wikipedia . ¿Quién está en lo correcto y quién está equivocado?

Tenga en cuenta que ( 3 ) tampoco es una regla de transformación correcta. Debería ser tu ( Λ ) | pag s = s D ( W ( Λ , pag ) ) s s | Λ pag , s , dónde W ( Λ , pag ) es la rotación de Wigner. Comuníquese con Weinberg, volumen 1, capítulo 2 para obtener más detalles.
@Blazej ¡Gracias! Realmente no veo la diferencia entre tu fórmula y la mía :-S acabas de escribir el D matriz con todo detalle, pero el contenido de nuestras dos fórmulas es el mismo, ¿verdad? (Escribí D σ σ ( Λ ) y escribiste D s s ( W ( Λ , pag ) ) , pero estos son los mismos, ¿no?).

Respuestas (2)

Podemos responder a esta pregunta de forma más general: ¿cuáles son las propiedades de transformación de las polarizaciones asociadas a un campo masivo? (Me estoy restringiendo a masivo porque las polarizaciones para partículas sin masa con espín mayor o igual que 1 requieren la discusión de la invariancia de calibre; lo dejaré para otro día).

Las polarizaciones se pueden definir sin referencia a la ecuación de campo: se definen como elementos de matriz para un campo entre el vacío y los estados de una partícula:

0 | ψ ( 0 ) | pag , σ tu σ ( pag )
donde el factor de proporcionalidad es una constante (conocida como renormalización de función de onda) en la normalización relativista para el estado | pag , σ . Aquí el índice es el índice en la representación de Lorentz llevado por el campo ψ (Por ejemplo, = m para un 4-vector, = α para un dirac spinor, más generalmente = ( α , β ) es un par de índices en el ( A , B ) representacion de S O ( 3 , 1 ) S tu ( 2 ) × S tu ( 2 ) ). Las polarizaciones llevan también otro índice, σ , que representa el espín de la partícula. Más precisamente, es el índice de grupo pequeño que lleva la partícula, ya sea el espín o la helicidad. Esta definición nos dice inmediatamente cómo tu σ se transforma dado que ψ lleva una representación de Lorentz
tu ( Λ ) ψ ( X ) tu 1 ( Λ ) = D ( Λ 1 ) ψ ( Λ X )
y la transformada de estado de una partícula con respecto al pequeño grupo con una rotación de Wigner
tu ( Λ ) | pag , σ = L σ σ ( W ( Λ ) , pag ) | pag Λ , σ
lo que implica
D ( Λ ) tu σ ( pag ) = tu σ ( pag Λ ) L σ σ ( W ( Λ ) , pag )
dónde pag Λ es parte de 3 vectores de 4 vectores Λ pag . (Una forma de leer esta ecuación es diciendo que las polarizaciones se transforman a la izquierda bajo Lorentz y a la derecha bajo las transformaciones de pequeños grupos: esto tiene que ser tal que uno pueda convertir los índices de Lorentz de funciones de correlación de campos en el pequeños índices de grupo de la matriz de dispersión S como dicta la fórmula de reducción LSZ).

Esto responde a tu pregunta. Pero, de hecho, podemos decir más: esas propiedades de transformación son constructivas ya que te permiten determinar explícitamente la polarización (y muestran que satisfacen ciertas ecuaciones, por ejemplo, Dirac para espín-1/2,...) como se demostró hace mucho tiempo. en los años 60 por Weinberg (ver la discusión en su libro de texto sobre QFT vol.1 capítulo 5). Por ejemplo, tome k = ( metro , 0 ) (para una partícula masiva) y aplicar la transformación canónica de Lorentz L = Λ eso lo trae a pag = ( mi , pag ) = L k . En este caso la rotación de Wigner es trivial, W = 1 , y por lo tanto

tu σ ( pag ) = D ( L ) tu σ ( 0 )
lo que significa que solo necesitamos conocerlos en momento cero (o, para partículas sin masa con respecto al vector de referencia utilizado para el pequeño grupo). Además, para una rotación arbitraria Λ = R tenemos W = R para cualquier pag de modo que
D ( R ) tu σ = tu σ ( 0 ) L σ σ ( R )
Tomando en diagonal las rotaciones alrededor del z eje, L σ σ ( R z ) = mi i σ θ d σ σ , la polarización se puede extraer por el infinitesimal z -rotación
D ( j z ) tu σ 0 = σ tu σ ( 0 ) .

Permítanme dar un ejemplo instructivo: un estado masivo spin-1 (donde es un índice en el irrep ( 1 / 2 , 1 / 2 ) , eso es = m = 0 , 1 , 2 , 3 es un índice de 4 vectores) tiene una representación tridimensional de j z dónde D ( j z ) i j = i ϵ 3 i j y D ( j z ) 00 = D ( j z ) i 0 = 0 para que las polarizaciones

ϵ m ± ( 0 ) = 1 2 ( 0 , 1 , 1 , 0 ) T , ϵ m 0 = ( 0 , 0 , 0 , 1 ) T .
resuelve las ecuaciones deseadas arriba. Claramente satisfacen pag m ϵ m σ = 0 , una vez que impulsamos ( metro , 0 ) a pag . Por lo tanto, el elemento de la matriz
Ψ m ( X ) 0 | ψ m ( X ) | pag , σ = mi i pag X 0 | ψ m ( 0 ) | pag , σ mi i pag X tu m σ ( pag )
satisface
( + metro 2 ) Ψ m ( X ) = 0 , m Ψ m ( X ) = 0
que se derivan más que se utilizan como punto de partida.

Se puede hacer lo mismo para cualquier espín, en particular para el espín-1/2 y ver que resuelven la ecuación de Dirac. De manera más general, dado que el grupo de Lorentz S O ( 3 , 1 ) S tu ( 2 ) A × S tu ( 2 ) B el momento angular viene dado por j = j A + j B lo que nos dice que

D ( j A ) α α tu α β σ ( 0 ) + D ( j B ) β β tu α β σ ( 0 ) = L ( j S ) σ σ tu α β σ ( 0 )
dónde = ( α , β ) . En otras palabras, las polarizaciones son (proporcionales a) el coeficiente de Clebsch Gordan para el espín S encontrado dentro A B
tu α β σ ( 0 ) C ( A B ) α β ( S ) σ
Varias de las propiedades de las polarizaciones provienen de hecho de la condición unitaria de estos coeficientes de Clebsch Gordan.

Entonces, para decirlo en términos simples, la respuesta corta es que esos enlaces en la parte inferior de mi publicación simplemente están equivocados. Alguien debería editar la entrada de wikipedia entonces, ¿verdad?
@AccidentalFourierTransform No, esos enlaces son correctos, por lo que es mi respuesta. Te estás perdiendo el punto que he tratado de enfatizar cuidadosamente: que las polarizaciones tienen dos índices, uno espinorial que se transforma así como en esos enlaces, y un índice de grupo pequeño que también se transforma, pero con las rotaciones de Wigner. No hay contradicción, esos enlaces simplemente discuten el caso de objetos con un solo índice espinorial (como, por ejemplo, un campo de espín 1/2) mientras que las polarizaciones tienen otro índice de grupo pequeño adicional. Incluso puedes ver que mi segunda ecuación está de acuerdo con lo que se afirma en los enlaces.
@AccidentalFourierTransform lo siento, solo miré la página wiki, que es correcta, como decía en el comentario que acabo de publicar. Por otro lado, la respuesta anterior en SEhttp://physics.stackexchange.com/questions/87575/unitary-lorentz-transformation-on-quantized-dirac-spinor/228078#228078 que estaba vinculando es simplemente incorrecta.

Primero, tenga en cuenta que u(ps)u(ps) no son estados en el espacio de Hilbert de alguna teoría cuántica. En cambio, son la solución a cierta ecuación, a saber ( γ m pag m metro ) tu ( pag s ) = 0 (equivalentemente: tu ( pag s ) mi i pag X resuelve la ecuación de Dirac). Por lo tanto, la notación de paréntesis no está realmente en su lugar (¡aunque es tentador!) La segunda observación es que debería pensar en cómo se define el giro. La convención habitual dice que el giro de una partícula en movimiento se define como su giro en el marco de referencia donde no se mueve. Ahora deja pag 0 = ( metro , 0 , 0 , 0 ) y definir tu ( pag 0 s ) como solución de la ecuación de Dirac que describe una partícula en reposo con espín s . Para cualquier rotación (que es solo la transformación de Lorentz Λ tal que Λ pag 0 = pag 0 ) tenemos una relación familiar de QM ordinario

S ( Λ ) tu ( pag 0 s ) = D ( Λ ) s s tu ( pag 0 s )
Luego, para cualquier impulso posible pag de esta partícula elegimos algún impulso estándar Λ 0 ( pag ) que transforma pag 0 a pag . La opción estándar es simplemente impulsar en el pag dirección. Ahora define tu ( pag s ) = S ( Λ 0 ( pag ) ) tu ( pag 0 s ) . Esta es una solución que describe una partícula en movimiento con espín. s en su marco de reposo. Ahora elijamos alguna transformación de Lorentz arbitraria Λ y calcular su acción
S ( Λ ) tu ( pag s ) = S ( Λ Λ 0 ( pag ) ) tu ( pag 0 s ) = S ( Λ 0 ( Λ pag ) ) S ( W ( Λ , pag ) ) tu ( pag 0 s ) ,
dónde W ( Λ , pag ) = Λ 0 ( Λ pag ) 1 Λ Λ 0 ( pag ) . Esta transformación de Lorentz se llama rotación de Wigner. Es fácil ver eso W ( Λ , pag ) pag 0 = pag 0 , por lo que se aplica la fórmula anterior. Por lo tanto
S ( Λ ) tu ( pag s ) = S ( Λ 0 ( Λ pag ) ) s D s s ( W ( Λ , pag ) ) tu ( pag 0 s ) = s D s s ( W ( Λ , pag ) ) tu ( Λ pag , s ) .
La segunda igualdad se sigue de la definición de tu ( pag s ) para pag pag 0 .

Entonces, en términos simples, estás diciendo que el s los componentes se mezclan bajo las transformaciones de Lorentz, ¿verdad? O dicho de otra manera, la fórmula correcta es S tu s = s D s s tu s , en lugar de S tu s = tu s , ¿bien?
tal vez debería haber escrito S tu s = s METRO s s tu s para alguna matriz METRO . Quería escribir la estructura general de la ecuación, sin preocuparme por los detalles. Para mí lo importante es que se mezclen, pero no que la matriz sea D o alguna otra matriz. De todos modos, editaré mi publicación en un minuto o dos...
Se mezclan pero no a través D ( Λ ) pero en lugar de a través D ( W ( Λ , pag ) ) . Tenga en cuenta varios hechos importantes. Primero, que esta rotación de Wigner depende explícitamente del momento de la partícula. Segundo, eso D ( Λ ) ni siquiera tendría sentido porque la matriz de Wigner D se define para rotaciones y no para transformaciones generales de Lorentz. En segundo lugar, usando estas fórmulas puedes convencerte de que si tomas una partícula en reposo con alguna configuración de espín, la impulsas una vez y luego otra vez en otra dirección, terminarás con algo con una polarización diferente.